Đến nội dung

Hình ảnh

Đề thi môn Toán chuyên tuyển sinh lớp 10 chuyên Trần Hưng Đạo Bình Thuận năm học 2019-2010


  • Please log in to reply
Chủ đề này có 27 trả lời

#1
nntien

nntien

    Sĩ quan

  • Thành viên
  • 372 Bài viết

Đề thi môn Toán chuyên tuyển sinh lớp 10 chuyên Trần Hưng Đạo Bình Thuận năm học 2019-2010

Hình gửi kèm

  • toan-chuyen-binh-thuan-result.jpg

$Maths$$Smart Home$ and $Penjing$

123 Phạm Thị Ngư


#2
Sin99

Sin99

    Thượng sĩ

  • Thành viên
  • 238 Bài viết

Câu 3. Đặt $ x = \frac{1}{a}, y = \frac{1}{b}, z = \frac{1}{c}  $. Ta có $ abc = 2 $ 

VT = $ \sum \frac{\frac{1}{x^2}}{\frac{1}{y}+ \frac{1}{z}} = \sum \frac{a^2}{b+c} \geq \frac{(a+b+c)^2}{2(a+b+c)} = \frac{a+b+c}{2} = \frac{a+b+c}{abc} = \sum \frac{1}{ab} = \sum xy  $ = VP. 


Bài viết đã được chỉnh sửa nội dung bởi Sin99: 14-06-2019 - 18:23


#3
Sin99

Sin99

    Thượng sĩ

  • Thành viên
  • 238 Bài viết

Câu 2a: $ (n+1)^4 + n^4 + 1 = 2(n^2 + n+1) $ 



#4
Sin99

Sin99

    Thượng sĩ

  • Thành viên
  • 238 Bài viết

Câu 4a: Ta có: $ FG^2 = FB.FA $ 

Mặt khác,  $ \angle FEC  = \angle ECD = \angle EAF $ nên EBF đồng dạng AEF (g-g) suy ra $ FE^2 = FB.FA $

Vậy $ FG = FE $



#5
Sin99

Sin99

    Thượng sĩ

  • Thành viên
  • 238 Bài viết

4b) Gọi L là trung điểm BI, M là giao điểm AH và BJ.

Ta có ABI đồng dạng AIJ (g-g) suy ra ABL đồng dạng AIH (c-g-c) => $ \angle BAL = \angle IAH $ 

Ta sẽ chứng minh $ \angle BAL = \angle CBJ $ 

Thật vậy, ta có IJC đồng dạng AIC mà tg AIC = tg AIB nên IJC đồng dạng AIB. 

Suy ra $ \frac{IC}{AB} = \frac{JC}{IB} $ hay $ \frac{2.IC}{AB} = \frac{2.JC}{IB} $ $ \Leftrightarrow \frac{BC}{AB} = \frac{JC}{BL} $ 

mà $ \angle B = \angle C $ nên tg ABL đồng dạng BCJ suy ra  $ \angle BAL = \angle CBJ $, kết hợp với  $ \angle BAL = \angle IAH $ ta có AMIB nội tiếp. 

Vậy $ \angle AMB = 90 $. 

Hình gửi kèm

  • faffsf.PNG

Bài viết đã được chỉnh sửa nội dung bởi Sin99: 14-06-2019 - 20:07


#6
Eugeo Synthesis 32

Eugeo Synthesis 32

    Lính mới

  • Thành viên mới
  • 4 Bài viết

Bài 5 thực ra giống y câu 21 đề thi MYTS lớp 8 năm 2018 nhé các bạn, chỉ thế buổi tiệc thành tuyên dương và chỉ thay bạn Hà thành An thoy =))))))))))))))      Đáp án là 14 nhé các bạn! Bạn nào muốn xem đề thi này thì tham khảo đề MYTS bằng cách bấm giô link này nhé =)))))))))))))))))))     ! https://drive.google...F9yA05srH8oQ0Pw



#7
Kim Shiny

Kim Shiny

    Binh nhất

  • Thành viên mới
  • 39 Bài viết

Bài 5 thực ra giống y câu 21 đề thi MYTS lớp 8 năm 2018 nhé các bạn, chỉ thế buổi tiệc thành tuyên dương và chỉ thay bạn Hà thành An thoy =))))))))))))))      Đáp án là 14 nhé các bạn! Bạn nào muốn xem đề thi này thì tham khảo đề MYTS bằng cách bấm giô link này nhé =)))))))))))))))))))     ! https://drive.google...F9yA05srH8oQ0Pw

có bạn nào giải được câu này thì chỉ mk vs ạ



#8
toanhoc2017

toanhoc2017

    Thiếu úy

  • Banned
  • 628 Bài viết

Bổ đề: các đa giác đều có số lẻ cạnh đều là tập "cân bằng" và "không tâm".
Chứng minh: Gọi đa giác đều trên là $A_1A_2...A_{2k+1}$. Gọi $(O)$ là đường tròn ngoại tiếp đa giác đều trên. Gọi đường kính đi qua tâm và 1 đỉnh của đa giác đều trên là "trục" của đa giác đều thì ta luôn có $2k+1$ trục như vậy. xét các cạnh $A_{i}A_1,A_{i}A_2,...,A_{i}A_{i-1},A_{i+1},...,A_{i}A_{2i+1}$, vì qua $A_{i}$ có 1 trục của đa giác đều nên trong số $2i$ cạnh kia có thể chia thành $i$ cặp cạnh bằng nhau với các độ dài phân biệt không bằng nhau. Từ đấy suy ra không tồn tại 3 cạnh chung đỉnh nào của đa giác đều bằng nhau Vậy ta chứng minh được bổ đề.
a). Từ các $2k+1$-giác đều, ta dựng tâm của các đa giác đấy. Khi đó, ta được một tập "cân bằng" có số chẵn điểm. Kết hợp với các $2k+1$-giác đều, ta có đpcm.
b). Xét 1 tập cân bằng $A$ có $2k$ điểm $A_1,A_2,A_3,...A_{2k}$. 1 điểm $A_i$ gọi là tốt đối với 1 cặp $A_{k},A_{t}$ ($k \neq i \neq t $) nếu $A_{i}A_{k}=A_{i}A_{t}$. Có tất cả $C_{2k}^{2}=k(2k-1)$ cặp điểm, mà trong $S$ có $2k$ điểm nên theo Dirichlet thì phải tồn tại 1 điểm $A_i$ tốt với $k$ cặp điểm. Nhưng vì trong $k$ cặp này không có cặp chứa điểm $A_i$ nên sẽ phải có 2 cặp $(A_t,A_{t_1})$ và $(A_t,A_{t_2})$ có trùng 1 điểm $A_t$.Khi đó ta có $A_i=A_{t_1}=A_{t_2}$, suy ra tập $S$ có $2k$ phần tử không phải là tập không tâm. Kết hợp với bổ đề, với các số tự nhiên $n$ lẻ lớn hơn $1$ thì luôn tồn tại $S$ có $n$ phần tử sao cho tồn tại một tập vừa cân bằng, vừa không tâm và có $n$ điểm.  

 

Bài 2. 
Xác định tất cả bộ ba $(a,b,c)$ các số nguyên dương sao cho các số sau

$ab-c,\text{ }bc-a,\text{ }ca-b$

đều là các lũy thừa của $2.$ 
Câu 2: Theo đề bài, ta đặt $ab-c=2^x, bc-a=2^y,ca-b=2^z$ , giả sử $x \ge y\ ge z$, từ đó ta có $b \ge a \ge c$. Với $y=z$ dễ dàng tìm ra bộ nghiệm $(2,2,2)$. Xét TH $y>z$, giả sử $c > 4$ .
Đầu tiên, ta sẽ chứng minh $3a < b$. Giả sử như $3a > b $ thì $-3a<-b \Leftrightarrow a(c-3) < 2^{z}$ hay $a < 2^{z-1}$.
Ta có: 
$\begin{cases}ab-c-bc+a=2^z(2^{x-y}-1)\\ab-c+bc-a=2^z(2^{x+y}+1)\end{cases} \\
\Leftrightarrow \begin{cases}(b+1)(a-c)=2^y(2^{x-y}-1) (1)\\(b-1)(c+a)=2^y(2^{x-y}+1)(2) \end{cases}$(*)
Từ (*) ta thấy nếu $b+1$ chia hết cho 4 thì $b-1$ không chia hết cho 4, hay $c+a$ chia hết cho $2^{y-1}$ hay $a \ge 2^{y-2} \ge 2^{z-1}$ (vô lý). Tương tự như vậy với trường hợp $b-1$ chia hết cho 4 và trường hợp cả $b-1$ lẫn $b+1$ không chia hết cho 4, ta được $3a<b$.Có:
$\begin{cases}bc-a-ca+b=2^z(2^{y-z}-1)\\bc-a+ca-b=2^z(2^{y-z}+1)\end{cases} \\
\Leftrightarrow \begin{cases}(c+1)(b-a)=2^z(2^{y-z}-1) (1)\\(c-1)(b+a)=2^z(2^{y-z}+1)(2) \end{cases}$ .
Lấy (1) chia (2), được: 
$\frac{b-a}{b+a}=\frac{2^{y-z}-1}{2^{y-z}+1}.\frac{c-1}{c+1} \ge \frac{2^{y-z}-1}{2^{y-z}+1}.\frac{1}{2} \\
\Leftrightarrow 2^{y-z} \le \frac{3b-a}{3a-b}$.
Mà $3a-b < 0$ nên $2^{y-z} < 0$ (vô lý). Vậy nên $c \le 4$.
Từ đó tìm ra được các bộ nghiệm ứng với $(a,b,c)$ là $(2,2,2), (2,3,2), (5,7,3), (6,11,2)$.  

Bài 3.

Cho tam giác $ABC$ nhọn với $AB>AC.$ Gọi $\Gamma $ là đường tròn ngoại tiếp tam giác, $H$ là trực tâm của tam giác và $F$ là chân đường cao kẻ từ đỉnh $A.$ Gọi $M$ là trung điểm của $BC$. Gọi $Q$ là điểm nằm trên $\Gamma $sao cho $\angle HQA=90^\circ $và gọi $K$ là điểm nằm trên $\Gamma $sao cho $\angle HKQ=90^\circ $. Giả sử các điểm $A,B,C,K$và $Q$ đều phân biệt và nằm trên $\Gamma $ theo thứ tự đó.
Chứng minh rằng đường tròn ngoại tiếp hai tam giác $KQH$ và $FKM$ tiếp xúc nhau



#9
toanhoc2017

toanhoc2017

    Thiếu úy

  • Banned
  • 628 Bài viết

C1:Mấu chốt là có tứ giác điều hòa do đường thẳng đậm là tiếp tuyến của đường tròn đậm.
Cụ thể như sau:
Ta có $\widehat{JHK}=\widehat{AQ'H}=\widehat{HH'K}$ nên $HM$ là tiếp tuyến của $(KHH').$ 
Khi đó $\widehat{HKM}=\widehat{FKH'}.$
Biến đổi góc ta được $\widehat{JKM}+\widehat{FMK}=90^0,$ đây chính là đpcm. 

 C2:Gọi W, N, P lần lượt là trung điểm của HA,HQ,HK. T là điểm đối xứng của Q qua W. Dễ thấy HT là tiếp tuyến của đường tròn (N), (PNH),(HFM). Sử dụng phép vị tự tâm H tỉ số 1/2 dễ dàng suy ra P,N,W nằm trện đường tròn Euler. Suy ra HT, FM, NP đồng quy tại J. Việc còn lại là chứng minh JK là tiếp tuyen (N). Suy ra đpcm  



#10
toanhoc2017

toanhoc2017

    Thiếu úy

  • Banned
  • 628 Bài viết

Câu 5: 
CHo $x=y=0$ thì được $f(f(0))=f(0)$. 
Thay $x=0, y=f(0)$, ta có $2f(0)=f(0)^2 \Rightarrow f(0)=0$ hoặc $f(0)=2$.
TH1: $f(0)=2$.
Thay $y$ bởi $x$, $x=0$ vào pt đầu, ta được: $f(f(x))=f(x)+2(x-1)$ (1)
Thay $x$ bởi $x-1$, $y=1$ vào pt đầu, ta được: $f(x+f(x)-1)=x+f(x)-1$ 
Thay $x$ bởi $x+f(x)-1$ vào (1), ta được: $f(x)=2-x$.
TH2: f(0)=0.
Giả sử $f(x) \neq x$ $\forall x \in \mathbb{R}$. 
Thay $y$ bởi $f(x)-x$, ta được: $f(x(f(x)-x))=(f(x)-x)f(x)$ (2)
Thay $y$ bởi $x-f(x)$, $x$ bởi $-x$, ta được: $f(-x(x-f(x)))=(x-f(x))f(-x)$ (3)
Từ (3) và (2) suy ra $f(x)=-f(-x)$ (*)
Thay $y$ bởi $-x$ vào pt đầu, ta được: $f(x)+f(-x^2)=x-xf(x)$ (4)
Thay $x$ bởi $-x$ vào pt (4), ta được: $f(-x)+f(-x^2)=-x+x(f-x)$ (5)
Từ (4) và (5) suy ra: $-f(x)+x-xf(x)=-f(-x)-x+xf(-x)$ (6)
Thay (*) vào (6), ta có: $-f(x)+x-xf(x)=f(x)-x-xf(x) \Leftrightarrow f(x)=x$ (vô lý).
Vậy: $f(x)=x$ $\forall x \in \mathbb{R}$.
Vậy nghiệm của pt hàm là : $f(x)=2-x$ và $f(x)=x$.  



#11
toanhoc2017

toanhoc2017

    Thiếu úy

  • Banned
  • 628 Bài viết

Bài toán vẫn đúng ở cả 2 chiều thuận và đảo:

 

(1) Nếu $AB=AC$ thì dễ thấy đpcm.

(2) Ta xét trường hợp $AB > AC$, trường hợp còn lại tương tự. Đặt $AB=c, BC=a, CA=b$ với $c > b$.

 

*Chiều đảo:

Giả sử tam giác $ABC$ vuông tại $A$.

Gọi $O$ là trung điểm $BC$ và $M$ là trung điểm cung $BC$ chứa $A$. Dễ thấy rằng $MB_1 = MC_1$.

Ta sẽ chứng minh rằng $MB_1 = MA_1$.

Thật vậy, ta có $BA_1 = \frac{a+b-c}{2}$ và $OB = \frac{a}{2}$ nên $OA_1 = \frac{c-b}{2}$. Do đó $4MA_1^2 = a^2+(b-c)^2$.

Đặt $H$ là hình chiếu của $M$ trên đoạn $AB$, đặt $MA = x$, ta có:

$MB^2 - MA^2 = BH^2 - AH^2$ hay $\frac{a^2}{2} - x^2 = c(c-\frac{x}{\sqrt{2}})$. Từ đó ta tính được $MA^2 = \frac{(b-c)^2}{2}$.

 

 

 

Theo định lí cos trong tam giác $MAC_1$ thì:

$MC_1^2 = MA^2+AC_1^2 - 2.MA.AC_1. \cos 45^{\circ} = \frac{(b-c)^2}{2} + \frac{(a+c-b)^2}{4}-2 . \frac{\sqrt{2}}{2}. \frac{a+c-b}{2}. \frac{c-b}{\sqrt{2}}$.

 

Từ đó, ta có được $MA_1^2 = MC_1^2$ hay $M$ chính là tâm đường tròn ngoại tiếp tam giác $A_1B_1C_1$. Ta có đpcm.

 

*Chiều thuận:

Tiếp theo, giả sử tâm $M$ của $(A_1B_1C_1)$ thuộc đường tròn $(ABC)$ và không mất tính tổng quát, có thể xét $M$ thuộc cung $BC$ chứa $A$ của $(ABC)$.

 

Ta có $MB_1 = MC_1$ và theo tính chất tiếp điểm của đường tròn bàng tiếp thì $BC_1 = CB_1$. Bằng biến đổi góc, ta cũng có $\widehat{ABM} = \widehat{ACM}$ nên ta có 2 trường hợp:

 

(1) $\Delta MB_1C = \Delta MC_1B (c.g.c)$ hoặc

(2) $\widehat{MC_1B} + \widehat{MB_1C}=180^{\circ}$ (không thể xảy ra)

 

Do đó, $MB=MC$ hay $M$ là trung điểm cung $BC$ chứa $A$ của $(ABC)$.

 

Đến đây, kẻ $MH, MK$ lần lượt vuông góc với $AB, AC$; theo giả sử thì $H$ thuộc đoạn $AB$ còn $K$ nằm ngoài đoạn $AC$ và theo tính chất đường phân giác, ta có $AH = AK$ hay

$AB - AM . \sin(\frac{180^{\circ} - {A}}{2}) = AC + AM . \sin(\frac{180^{\circ} - {A}}{2})$.

Biến đổi tương đương:

$AB - AC =2 AM. \cos \frac{A}{2}$

$\sin {C} - \sin { B} = 2 \sin \frac{C-B}{2} \cos \frac{A}{2}$.

$\tan \frac{A}{2} = 1$.

$A = 90^{\circ}$.

Ta có đpcm.

 

Vậy tâm đường tròn $(A_1B_1C_1)$ thuộc $(ABC)$ khi và chỉ khi tam giác $ABC$ vuông.



#12
toanhoc2017

toanhoc2017

    Thiếu úy

  • Banned
  • 628 Bài viết

Ta có $BC_1=CB_1=p-a $. Không mất tính tổng quát giả sử đường trung trực của $BC $ cắt (ABC) tại $O_1 $ thuộc cung BAC. Dễ thấy các tam giác $O_1BC_1 $và $O_1CB_1 $ bằng nhau (c.g.c) nên $O_1B_1=O_1C_1 $.

Theo kết quả quen thuộc $O_1A_1^2=O_1B^2-(p-b)(p-c) $ nên $O_1A_1^2-O_1B^2=\frac{(b-c)^2-a^2}{4} $

Gọi $K, L $ thứ tự là hình chiếu của $O_1 $ trên $CA, AB $. Các tam giác $O_1BL, O_1CK $ bằng nhau.

Dễ thấy $AK=AL $ nên ta có $BL=CK=\frac{b+c}{2} $

$O_1C_1^2-O_1B^2=LC_1^2-LB^2=(\frac{b+c}{2}-\frac{b+c-a}{2})^2-(\frac{b+c}{2})^2=\frac{a^2-(b+c)^2}{4} $

Đpcm tương đương $O_1 $ là tâm ngoại tiếp tam giác $A_1B_1C_1 $ hay $O_1A_1^2=O_1C_1^2 $

Tương đương với $O_1A_1^2-O_1B^2=O_1C_1^2-O_1B^2 $

$\frac{(b-c)^2-a^2}{4}=\frac{a^2-(b+c)^2}{4} $

hay$ a^2=b^2+c^2 $.

Bài toán được chứng minh xong. 

C2Trong (i), lấy $x=1$ và $y=a$, ta được
$$f(1)f(a) \ge f(a) \Rightarrow af(1) \ge a \Rightarrow f(1) \ge 1.$$
Từ (2) suy ra với $n\in\mathbb Z, n\ge1$, ta có
$f(nx) \ge nf(x),\,\forall x \in {\mathbb Q_{ > 0}}$, suy ra
\[f(n) \ge nf(1) \ge n \Rightarrow f(n) \ge n,\,\forall n \in {\mathbb N^*}.\tag{1}\]
Ta có $ f(q)f(\dfrac{p}{q})\mathop \ge \limits^{\text{do}\,\,(i)} f(p)\mathop \ge \limits^{\text{do}\,\,(1)} p\,\,\,(p,q \in {\mathbb N^*}).$ Từ 
đây suy ra 
\[f(x) > 0,\,\forall x \in {\mathbb Q_{ > 0}}.\tag{2}\]
Từ (2) và (ii) suy ra $f$ là hàm tăng nghiêm ngặt trên ${\mathbb Q_{ > 0}}$.
Với $q$ là số hữu tỉ, $q>1$, ta có $f(q) \ge f([q]) \ge [q] > q - 1$.

$\bullet$ Chứng minh: $ f(q)\ge q $, với $q$ là số hữu tỉ lớn hơn 1.
Giả sử tồn tại $q\in\mathbb Q$, $q>1$ sao cho $f(q)<q$, khi đó $ f(q) = q-\varepsilon $, với $0 < \varepsilon < 1$.
Ta có 
\[{(q - \varepsilon )^n} = f{(q)^n}\mathop \ge \limits^{\text{do}\,\,(i)} f({q^n}) > {q^n} - 1.\]
Do đó 
${\left( {\dfrac{{q - \varepsilon }}{q}} \right)^n} > 1 - \dfrac{1}{{{q^n}}},\,\forall n = 1,2,\dots$
Từ đây cho $n\to+\infty$, ta được $0\ge 1$, vô lí. Vậy với mọi 
$q$ là số hữu tỉ lớn hơn 1, ta có $ f(q)\ge q $.

$\bullet$ Chứng minh: $ f(q)= q $, với $q$ là số hữu tỉ, $q\ge 1$.
Chọn số nguyên dương $n$ đủ lớn sao cho $ a^n-q > 1 $. Khi đó
$${a^n} = f{(a)^n} \ge f({a^n}) = f({a^n} - q + q) \ge f({a^n} - q) + f(q) \ge {a^n} - q + q = {a^n}.$$
Suy ra $ f(q)= q $, với $q$ là số hữu tỉ, $q\ge 1$. (3)

$\bullet$ Giả sử $q \in {\mathbb Q_{ > 0}}$ và $q<1$. Khi đó $\dfrac{1}{q} > 1$. Ta có
$$f(q)f(\frac{1}{q}) \ge f(1) \Rightarrow f(q)\frac{1}{q} \ge f(1) \Rightarrow f(q)\frac{1}{q} \ge 1 \Rightarrow f(q) \ge q.$$
Vậy $f(q) \ge q,\,\forall q \in {\mathbb Q_{ > 0}}$.
Giả sử tồn tại số hữu tỉ $q\in(0;1)$ sao cho $f(q)>q$. Khi đó
\[f(nq)\mathop \ge \limits^{\text{do}\,\,(ii)} nf(q) > nq,\,\forall n = 2,3,\dots\tag{4}\]
Chọn số nguyên dương $n$ đủ lớn sao cho $nq>1$, khi đó từ (4) ta có $f(nq)>nq$, điều này mâu thuẫn với (3).
Do đó, với mọi số hữu tỉ $q\in(0;1)$, ta có $f(q)=q$. (5)

Từ (3) và (5) suy ra $f(x) = x,\,\forall x \in {\mathbb Q_{ > 0}}$, điều phải chứng minh.  

 

Dễ chứng minh $f(1) \ge 1$.
ta chứng minh:$(f(a))^2 \ge a^2.f(1)$. Thật vậy, từ $f(x).f(y)=f(x.y)$ (1), ta thay:$x=y=a$, được: $(f(a))^2 \ge f(a^2)$. Mà từ $f(x+y) \ge f(x)+f(y) $ (2), ta có: $f(a^2) \ge f(a^{2}-1) +f(1) \ge f(a^{2}-2)+2f(1) \ge ... \ge a^{2}.f(1)$ suy ra $(f(a))^2=a^2 \ge a^2.f(1) \Rightarrow f(1) \le 1 $. Mà ta đã chứng minh $f(1) \ge 1$ nên $f(1)=1$. Xét TH $a$ không nguyên, khi đó: $a^2=f(a)^2 \ge f(a^2) \ge f(a^2-1)+f(1) \ge a^2-1+1=a^2$ nên $f(1)=1$ . Từ đó dễ chứng minh bằng quy nạp rằng $f(n)=n$ với mọi $n \in \mathbb N*$. Giờ ta chứng minh $f(\frac{1}{n})=\frac{1}{n}$.
THật vậy: $1=f(1)=f(n.\frac{1}{n}) \le f(n).f(\frac{1}{n}) \Leftrightarrow f(\frac{1}{n}) \ge \frac{1}{n}$, tương tự, ta chứng minh được: $f(\frac{m}{n}) \ge \frac{m}{n}$. Mà $1=f(1)=f(\frac{n}{n}) \ge f(\frac{n-1}{n})+f(\frac{1}{n}) \ge f(\frac{n-2}{n}) + 2.f(\frac{1}{n}) \ge ... \ge n.f(\frac{1}{n}) \Rightarrow f(\frac{1}{n}) \le \frac{1}{n}$ mà ta cũng đã chứng minh $f(\frac{1}{n}) \ge \frac{1}{n}$ nên $f(\frac{1}{a})=\frac{1}{a}$ với mọi $n \in \mathbb N*$.
Giờ ta chứng minh $f(\frac{m}{n})=\frac{m}{n}$.
Ta có: $f(\frac{m}{n}). f(\frac{n}{m}) \ge f(1) $, mà $f(\frac{m}{n}). f(\frac{n}{m}) \le f(m).f(\frac{1}{n}).f(n).f(\frac{1}{m})= 1$. Từ đấy suy ra $f(\frac{m}{n}). f(\frac{n}{m})=1$, mà $f(\frac{m}{n}) \ge \frac{m}{n}, f(\frac{n}{m}) \ge \frac{n}{m}$ nên $f(\frac{m}{n})= \frac{n}{m}, f(\frac{n}{m}) = \frac{n}{m}$ với mọi $n;m \in \mathbb N*$.  

Bổ đề: Giả sử có cách đánh số sao cho $a_0 = 0, a_1 = a, a_n = n $ thì ta phải có $(a,n) = 1 $ và $a_k \equiv ka \pmod n $ và cách đánh số đó là duy nhất.

Chứng minh: ta chứng minh bằng quy nạp theo $k $ rằng $a_k\equiv ka \pmod n $.
Ta có $a_1 = a $, nên khẳng định đúng với $k = 1 $. Ta chứng minh với $k + 1 $.
Giả sử $a_{k+1} = x \neq (k+1)a\pmod n $ ta xét các trường hợp sau:

Th1: $x > a $, khi đó ta phải có $x-a $ phải thuộc tập $\{a_1,...,a_k\} $ hay $x - a \equiv la \pmod n $ với $0\le l\le k $, mà $a_{k+1}\neq la \pmod n $ với mọi $0\le l\le k $ nên vô lý.

Th2: $x < a $, khi đó ta có $n-a+x < n $ và dễ thấy vì $(n - a + x + a) = x + n $ nên ta cũng phải có $n-a +x $ xuất hiện trước $x $, hay $n-a+x = la\pmod n $ với $1\le l\le n $, suy ra $x\quiv (l+1)\pmod n $ (mâu thuẫn).

Từ điều trên ta dễ thấy là phải có $(a,n) = 1 $.

Vậy bổ đề được chứng minh.


Trở lại bài toán:

Với mỗi cách đánh số thỏa mãn bài toán đặt $a_0 = 0, a_1 = a, a_n = b $. Giả sử $a < b $ (trường hợp $a > b $ tương tự). Nếu $b = n $ theo bổ đề ta có $(a,n) = 1 $ và với mỗi cặp $(a,n) = 1 $ thì có đúng một cách đánh số thỏa mãn.

Nếu $b < n $, dễ thấy là ta phải có $a + b > n $. Do đã giả sử $b > a $ nên ta có $L = n-b < a $. Ta bỏ đi $L < a $ số $b+1,...n $, khi đó ta thu được cách đánh số thỏa mãn với cặp đỉnh kề $ 0 $ là $a $ và $b $ và max là $b $. Theo bổ đề ta có duy nhất một cách đánh số thỏa mãn với $(a,b) = 1 $ là $0,a \pmod b,2a \pmod b,...,b-a \pmod b, b $. Bây giờ ta chèn lại các số $b+1,...n $ vào cách đánh số này.
Dễ thấy là các số $1,2,..,L $ lần lượt đi liền sau các số $b-a+1,b-a+2,...,b-a+L $ (chú ý là $l\le n-b = L < a $ và hiệu của hai số liền nhau của cách đánh số cho $(a,b) = $1 là $a $ hoặc $a-b $). Đến đây ta phải chèn số $b+l $ vào giữa $b-a+l $ và $l $. Cách chèn này là duy nhất. 

Mặt khác có thể chứng minh rằng cách chèn lại các số $b+1,...,n $ sẽ thu được cách đánh số thỏa mãn với $n $ số. (Không khó lắm )

Tổng hợp ta có số cách đánh số thỏa mãn chính bằng số bộ $(a,b) = 1 $ mà $a + b > n, 1\le a,b\le n $. Dễ dàng chứng minh được số này = $1 + \sum\limits_{k=2}^{n} \varphi(k) $. ĐPCM.  

C4Em có cách này chắc khá giống cách anh Traum nhưng vẫn post lên để mọi người xem có thiếu gì không( vì thấy có vẻ dễ nghĩ hơn và không xét chẵn lẻ).
Xét đường thẳng đi qua 2 trong số $2N+1 $ điểm sao cho tất cả $2N-1 $ điểm còn lại thuộc cùng một nửa mặt phẳng.
Theo giả thiết quy nạp thì $2N-1 $ điểm còn lại đã được chia thỏa mãn bằng $N-1 $ đường thẳng.
Xét các trường hợp:
TH1: 2 điểm thuộc đường thẳng thuộc 2 miền khác nhau khi chia bởi $N-1 $ đường thẳng kia 
a) 2 điểm cùng màu, thì điểm mà nằm trong miền khác màu với nó thì ta luôn kẻ được 1 đường thẳng chia mặt phẳng thành 2 nửa mặt phẳng mà một nửa mặt phẳng chứa điểm khác màu kia và một nửa chứa $2N $ điểm còn lại, đây chính là đường thẳng thứ $N $ chia các điểm theo đúng yêu cầu
b) 2 điểm khác màu mà cả hai đều nằm trong miền khác màu với chúng. Ta vẽ thêm đường thẳng song song với đường thẳng đi qua 2 điểm này và chia mặt phẳng thành 2 nửa, một nửa chứa 2 điểm, 1 nửa chưa $2N-1 $ điểm.
TH2: 2 điểm thuộc đường thẳng đều thuộc cùng 1 miền 
a) 2 điểm cùng màu, khác màu với miền chứa chúng, ta làm như TH 1b
b) 2 điểm khác màu, ta làm như TH 1a với điểm khác màu với miền trong 2 điểm đó
C5Xét đường gấp khúc không tự cắt đi qua mọi điểm của hệ.
Rõ ràng là số cạnh nằm trên đường gấp khúc sao cho hai điểm mút khác màu là không quá $n+2$.
Xét một dãy đỉnh liên tiếp cùng màu trên đường gấp khúc, ta có nếu số đỉnh liên tiếp này nhiều hơn 1 thì ta chỉ cần 2 đường thẳng để chia dãy đỉnh này với phần còn lại của đường gấp khúc(bằng cách chọn hai điểm nằm trên hai đoạn thẳng nối hai câp đỉnh khác màu liên tiếp ở ngoài cùng, rồi ta quay đường thẳng đó cho đến khi thoả mãn, và rõ ràng là tồn tại vì đường gấp khúc đã cho không tự cắt)

Xét hệ điểm đã cho, vì số điểm xanh và số điểm đỏ không bằng nhau nên ta có một dãy đỉnh cùng màu không ít hơn hai đỉnh. Nếu trong hệ điểm tồn tại một dãy đỉnh có không ít hơn 3 đỉnh liên tiếp cùng màu thì từ thuật toán trên đã có thể dùng hai đường thẳng để tách mặt phẳng đó ra thành 4 phần, trong đó có 1 phần là dãy đỉnh đã cho. Vì ta có $2n+1$ và $n$ đường thẳng nên sau khi chia ta chỉ cần xử lí không quá $2n-2$ với $n-2$ đường thẳng. Theo quy nạp thì mỗi hệ điểm có $k$ điểm chỉ cần không quá $\floor{\frac{n}{2}}$ đường thẳng. Do đó ta chỉ cần chia sao cho có 1 phần chứa số lẻ điểm và điều này có thể thực hiện được bằng cách quay đường thẳng.

Với trường hợp còn lại, chọn ba đỉnh liên tiếp sao cho ba đỉnh đó nằm ngoài bao lồi của hệ điểm còn lại(luôn chọn được), dùng 1 đường thẳng để tách 3 điểm đó ra và 1 đường thẳng để tách các điểm khác màu trong 3 điểm đó(luôn có do không tồn tại dãy 3 đỉnh liên tiếp cùng màu), rồi ta quay đường thẳng thứ 2 sao cho nó chia hệ điểm kia thành 1 phần có lẻ đỉnh, như vậy ta có dpcm.
Các trường hợp còn lại đều thỏa mãn với $N-1 $đường thẳng 



#13
toanhoc2017

toanhoc2017

    Thiếu úy

  • Banned
  • 628 Bài viết
Đề thi IMO 2012 và Lời giải

Ngày thi thứ nhất (10/7/2012)

Bài 1.

Cho tam giác $ABC$ và điểm $J$ là tâm đường tròn bàng tiếp trong góc $A$ của tam giác. Đường tròn này tiếp xúc với $AB,AC,BC$ tại $K,L,M$ theo thứ tự. $LM$ cắt $BJ$ tại $F$, $KM$ cắt $CJ$ tại $G$. Gọi $S,T$ lần lượt là giao điểm của $AF,AG$ với $BC$. Chứng minh rằng $M$ là trung điểm của $ST$.

Bài này không khó
Ta có $\triangle FKJ=\triangle FMJ$, do đó $\widehat{KFJ}=\widehat{JFM}$.
Mặt khác $\widehat{JFM}=\widehat{KAJ}$ nên $AFKJ$ nội tiếp. Suy ra $FJ\perp AS$. Từ đó $FS=FA$
Tương tự $AT$ vuông góc với $GJ$. Từ đó $AT//FM$.
Vậy $M$ là trung điểm của $ST$.

Bài 2.

Cho số nguyên $n \ge 3$ và các số thực dương $a_2,a_3,\ldots,a_n$ thỏa mãn $a_2 \cdots a_n= 1$. Chứng minh rằng
$$ (1+a_2)^2(1+a_3)^3 \cdots (1+a_n)^n > n^n. $$

 

Câu này khá dễ:Theo AM - GM ta có $ (a_{k}+1)=\left(a_{k}+\frac {1}{k-1}+\cdots+\frac {1}{k-1}\right)\geq k\sqrt[k]{\frac{a_{k}}{(k-1)^{k-1}}}; $.
Suy ra
$ (a_{k}+1)^{k}\geq\frac{k^{k}}{(k-1)^{k-1}}\cdot a_{k}. $
Do đó
\[ \prod_{k=2}^{n}(a_{k}+1)^{k}\geq n^{n}a_{2}a_{3}\cdots a_{n}=n^{n}. \]
Dấu bằng xảy ra nếu và chỉ nếu $ a_{k}=\frac {1}{k-1} $, vô lí.


Bài 3.

Trò chơi đoán kẻ nói dối là một trò chơi giữa hai người chơi $A$ và $B$. Quy tắc của trò chơi phụ thuộc vào hai số nguyên dương $k$ và $n$ mà cả hai người chơi đều đã biết trước.

Bắt đầu trò chơi, $A$ sẽ chọn các số nguyên $x$ và $N$ với $1 \le x \le N$. $A$ giữ bí mật số $x$ và nói số $N$ cho $B$. $B$ sẽ cố thu nhận thông tin về số $x$ bằng cách hỏi $A$ các câu hỏi như sau : mỗi câu hỏi bao gồm việc $B$ xác định một tập $S$ tùy ý các số nguyên dương (có thể là một tập đã được nhắc đến trong câu hỏi trước đó) và hỏi $A$ xem $x$ có thuộc $S$ hay không. Sau mỗi câu hỏi, $A$ phải trả lời hoặc không, nhưng có thể nói dối bao nhiêu lần tùy thích, chỉ có điều là phải trả lời đúng ít nhất một trong số $k+1$ câu hỏi liên tiếp.

Sau khi $B$ đã hỏi xong, $B$ phải chỉ ra một tập $X$ có tối đa $n$ số nguyên dương. Nếu $x \in X$, $B$ thắng; nếu ngược lại, $B$ thua. Chứng minh rằng :

  1. Nếu $n \ge 2^k$, $B$ có thể đảm bảo một chiến thắng.
  2. Với mọi $k$ đủ lớn, tồn tại một số nguyên $n \ge 1.99^k$ sao cho $B$ không thể đảm bảo có một chiến thắng.

Đây là bài khó.

 

**Ngày thi thứ hai (11/7/2012).

Bài 4.

Tìm tất cả các hàm số $f : \mathbb{Z} \to \mathbb{Z}$ sao cho với mọi $a+b+c=0$ thì
$$ f(a)^2+f(b)^2+f(c)^2 = 2f(a)f(b) + 2f(b)f(c) + 2f(c)f(a). $$

Bài 5.

Cho tam giác $ABC$ có $\widehat{ACB} = 90^\circ$ và $D$ là chân đường cao tương ứng với đỉnh $C$. Gọi $X$ là một điểm trong của đoạn thẳng $CD$. Gọi $K$ là điểm trên đoạn thẳng $AX$ sao cho $BK=BC$. Tương tự, gọi $L$ là điểm trên đoạn thẳng $BX$ sao cho $AL=AC$. Gọi $M$ là giao điểm của $AL$ và $BK$. Chứng minh rằng $MK=ML$.

Bài 6.

Tìm tất cả các số nguyên dương $n$ sao cho tồn tại các số nguyên không âm $a_1,a_2,\ldots,a_n$ thỏa mãn
$$ \frac{1}{2^{a_1}} + \frac{1}{2^{a_2}} + \cdots + \frac{1}{2^{a_n}} = \frac{1}{3^{a_1}} + \frac{2}{3^{a_2}} + \cdots + \frac{n}{3^{a_n}} = 1 $$

 



#14
toanhoc2017

toanhoc2017

    Thiếu úy

  • Banned
  • 628 Bài viết

Cho tam giác $ABC$ và điểm $J$ là tâm đường tròn bàng tiếp trong góc $A$ của tam giác. Đường tròn này tiếp xúc với $AB,AC,BC$ tại $K,L,M$ theo thứ tự. $LM$ cắt $BJ$ tại $F$, $KM$ cắt $CJ$ tại $G$. Gọi $S,T$ lần lượt là giao điểm của $AF,AG$ với $BC$. Chứng minh rằng $M$ là trung điểm của $ST$. 

Trò chơi đoán kẻ nói dối là một trò chơi giữa hai người chơi $A$ và $B$. Quy tắc của trò chơi phụ thuộc vào hai số nguyên dương $k$ và $n$ mà cả hai người chơi đều đã biết trước.

Bắt đầu trò chơi, $A$ sẽ chọn các số nguyên $x$ và $N$ với $1 \le x \le N$. $A$ giữ bí mật số $x$ và nói số $N$ cho $B$. $B$ sẽ cố thu nhận thông tin về số $x$ bằng cách hỏi $A$ các câu hỏi như sau : mỗi câu hỏi bao gồm việc $B$ xác định một tập $S$ tùy ý các số nguyên dương (có thể là một tập đã được nhắc đến trong câu hỏi trước đó) và hỏi $A$ xem $x$ có thuộc $S$ hay không. Sau mỗi câu hỏi, $A$ phải trả lời  hoặc không, nhưng có thể nói dối bao nhiêu lần tùy thích, chỉ có điều là phải trả lời đúng ít nhất một trong số $k+1$ câu hỏi liên tiếp.

Sau khi $B$ đã hỏi xong, $B$ phải chỉ ra một tập $X$ có tối đa $n$ số nguyên dương. Nếu $x \in X$, $B$ thắng; nếu ngược lại, $B$ thua. Chứng minh rằng :

  1. Nếu $n \ge 2^k$, $B$ có thể đảm bảo một chiến thắng.
  2. Với mọi $k$ đủ lớn, tồn tại một số nguyên $n \ge 1.99^k$ sao cho $B$ không thể đảm bảo có một chiến thắng.

 Tìm tất cả các hàm số $f : \mathbb{Z} \to \mathbb{Z}$ sao cho với mọi $a+b+c=0$ thì
$$ f(a)^2+f(b)^2+f(c)^2 = 2f(a)f(b) + 2f(b)f(c) + 2f(c)f(a). $$  

 

Cho tam giác $ABC$ có $\widehat{ACB} = 90^\circ$ và $D$ là chân đường cao tương ứng với đỉnh $C$. Gọi $X$ là một điểm trong của đoạn thẳng $CD$. Gọi $K$ là điểm trên đoạn thẳng $AX$ sao cho $BK=BC$. Tương tự, gọi $L$ là điểm trên đoạn thẳng $BX$ sao cho $AL=AC$. Gọi $M$ là giao điểm của $AL$ và $BK$. Chứng minh rằng $MK=ML$.  

__________________

Tìm tất cả các số nguyên dương $n$ sao cho tồn tại các số nguyên không âm $a_1,a_2,\ldots,a_n$ thỏa mãn
$$ \frac{1}{2^{a_1}} + \frac{1}{2^{a_2}} + \cdots + \frac{1}{2^{a_n}} = \frac{1}{3^{a_1}} + \frac{2}{3^{a_2}} + \cdots + \frac{n}{3^{a_n}} = 1 $$  



#15
toanhoc2017

toanhoc2017

    Thiếu úy

  • Banned
  • 628 Bài viết

Giả sử đã có $x \in S = \{1, ..., n\}$ với $n > 2^k$, ta chứng minh rằng sau một số câu hỏi, có thể tìm được tập con $S' \subset S$ sao cho $|S'| < |S|$ và chắc chắn $x \in S'$. Quá trình này sẽ dừng khi $|S| \leq 2^k$, và do đó suy ra được điều cần chứng minh.

 

Thật vậy, vì $|S| > 2^k$ nên tồn tại song ánh $f$ giữa một tập con thật sự $T$ của $S$ và tập hợp các chuỗi nhị phân độ dài $k$. Gọi $m$ là một phần tử nằm trong $S$ nhưng ko nằm trong $T$. Đồng thời ký hiệu $T_i$ là nghịch ảnh của tập hợp các chuỗi nhị phân có bit ở vị trí $i$-th là $0$, đối với $f$. Ta thực hiện các câu hỏi sau:

 

1. Hỏi liên tục các câu với tập hợp $\{m\}$. Nếu sau $k + 1$ câu hỏi như vậy mà ta đều nhận được đáp án "có" thì vì trong các câu này phải có ít nhất 1 câu $A$ nói thật nên có thể kết luận $X = \{m\}$, hết chuyện.

 

2. Ngược lại, giả sử tại câu hỏi thứ $i$-th ta có câu trả lời "không". Từ câu $i + 1$ đến câu $i + k$, ta lần lượt hỏi với các tập hợp $T_i$ như đã mô tả ở trên. Vì $i...i + k$ là $k + 1$ câu hỏi liên tiếp nên trong số chúng, có ít nhất 1 câu đáng tin. Thế tức là nếu ta kết hợp tất cả các điều kiện ngược với các câu trả lời thì dựa vào đó, tập hợp thu được sẽ vẫn chứa $x$. Ký hiệu $res(T_i) = T_i$ nếu câu trả lời ứng với câu hỏi $T_i$ là "có" và $res(T_i) = \overline{T_i}$ nếu câu trả lời này là "không" (ở đây $\overline{T_i}$ là tập hợp các chuỗi nhị phân độ dài $k$ và có bit ở vị trí $i$-th là $1$, hợp với các phần tử còn lại trong $S$, ngoài $m$ và các phần tử trong các $T_j$). Ta tìm giao của các tập hợp $\overline{res(T_i)} \cap (\bigcup T_i)$. Rõ ràng mỗi tập hợp kiểu này cho ta một mô tả về phần tử $x$ nằm trong giao của chúng (i.e. $x$ có bit thứ $i$-th là $0$ hay $1$). Vậy nên giao này khác rỗng (và tất nhiên là không phủ hết $(\bigcup T_i)$, vì bản thân các $\overline{res(T_i)}$ cũng không phủ được!). Do vậy, phần tử $x$ cần tìm mất đi ít nhất $1$ khả năng, và đó là điều ta đang muốn  

 

 

Bài 1.

Cho tam giác $ABC$ và điểm $J$ là tâm đường tròn bàng tiếp trong góc $A$ của tam giác. Đường tròn này tiếp xúc với $AB,AC,BC$ tại $K,L,M$ theo thứ tự. $LM$ cắt $BJ$ tại $F$, $KM$ cắt $CJ$ tại $G$. Gọi $S,T$ lần lượt là giao điểm của $AF,AG$ với $BC$. Chứng minh rằng $M$ là trung điểm của $ST$.

Bài này không khó
C1Ta có $\triangle FKJ=\triangle FMJ$, do đó $\widehat{KFJ}=\widehat{JFM}$.
Mặt khác $\widehat{JFM}=\widehat{KAJ}$ nên $AFKJ$ nội tiếp. Suy ra $FJ\perp AS$. Từ đó $FS=FA$
Tương tự $AT$ vuông góc với $GJ$. Từ đó $AT//FM$.
Vậy $M$ là trung điểm của $ST$.

C2Hạ $BP\perp AX, AQ\perp BX $.
Gọi $S $ là giao điểm của $AQ $ và $BP $.
Vì $X $ là trực tâm tam giác $SAB $ nên $SX\perp AB \Rightarrow S\in CD $.
Ta có$ AL^2=AC^2=AD.AH=AQ.AS\Rightarrow \widehat{ALS}=90^0 $
$\Rightarrow SL^2=SQ.SA $
Tương tự $SK^2=SP.SB $
Từ đó $SL=SK $ mà $\widehat{MLS}=90^0=\widehat{MKS}=90^0 $ nên $ML=MK $.

 

  C3Trước hết, ta có bổ đề sau :
Cho tam giác $ABG$ trực tâm $X$, các đường cao $GD,AE,BF$. Các đường tròn đường kính $GB,GA$ cắt $AX,BX$ tại $K,L$ theo thứ tự. Khi đó $GK=GL$.

Chứng minh bổ đề.
Ta có
$$ GK^2 = GE \cdot GB = GF \cdot GA = GL^2. $$
Suy ra $GK = GL$.

Trở lại với bài toán.

Gọi $G$ là trực tâm tam giác $AXB$; $K',L'$ là giao điểm của các đường tròn đường kính $GB,GA$ với $AX,BX$ theo thứ tự.
Từ bổ đề trên dễ dàng suy ra $K \equiv K', L \equiv L'$ và $GK=GL$. Mà tứ giác $GKML$ nội tiếp đường tròn đường kính $GM$ nên ta có điều cần chứng minh.  

 



#16
toanhoc2017

toanhoc2017

    Thiếu úy

  • Banned
  • 628 Bài viết

Bài 1. Ngon.
$s_A=a_1+a_2+a_3+a_4 $Có tất cả 6 cặp $a_i+a_j=a_1+a_2,a_2+a_3,a_3+a_4,a_1+a_4,a_2+a_3,a_ 2+a_4 $
Do 4 số nguyên dương phân biệt nên mình sắp lại thứ tự có $a_1<a_2<a_3<a_4 $.Khi đó $a_3+a_4|s_A $ khi $a_3+a_4|a_1+a_2 $ vô lý,cái $a_2+a_4 $ cũng thế.
Do vậy $n_A\leq 4 $
Có hệ $\begin{cases}
& a_1+a_2|s_A=a_1+a_2+a_3+a_4 \\ 
& a_1+a_3|s_A=a_1+a_2+a_3+a_4\\ 
& a_1+a_4|s_A=a_1+a_2+a_3+a_4\\ 
& a_2+a_3|s_A=a_1+a_2+a_3+a_4
\end{cases}
\rightarrow
\begin{cases}
& a_1+a_2|a_3+a_4 \\ 
& a_1+a_3|a_2+a_4\\ 
& a_1+a_4|a_2+a_3\\ 
& a_2+a_3|a_1+a_4
\end{cases}
\rightarrow
\begin{cases}
& a_1+a_2|a_3+a_4 \\ 
& a_1+a_3|a_2+a_4\\ 
& a_1+a_4=a_2+a_3\\ 
& a_1<a_2<a_3<a_4\\ 
\end{cases}
$

Cái này dễ giải ra tính được 4 số theo 1 số còn lại.Mathlinks ra đáp số ($x,5x,7x,11x $) và $(x,11x,19x,29x) $  

C1Bài 3:

[M]f(x+y) \le yf(x)+f(f(x))[/M] (1)
Thay [M]y=0[/M] ta có : [M]f(x) \le f(f(x))[/M]
Kí hiệu [M]D_f[/M] là tập giá trị của [M]f [/M]. Với [M]x \in D_f[/M], thay [M]y= f(x)-x[/M] không âm ta có [M]f(f(x)) \le yf(x)+f(f(x))[/M] nên [M]f(x)[/M] không âm với [M]x \in D_f[/M] nói cách khác [M]f(f(x))[/M] không âm với mọi [M]x [/M].
Bây giờ, giả sử tồn tại [M]x[/M] mà [M]f(x)>0 [/M], cho y tiến tới âm vô cùng thì sẽ tồn tại [M]e[/M] mà [M]f(x)<0[/M] với mọi [M]x \le e[/M]
tương tự nếu có [M]f(x)<0[/M] thì [M]f(x)<0[/M] với mọi [M]x[/M] đủ lớn.
Chú ý rằng nếu [M]f(x)<0[/M] thì [M]x[/M] không thuộc tập giá trị của [M]f[/M]
TH1: [M]f[/M] nhận cả giá trị âm dương. Thì [M]D_f[/M] là tập bị chặn . Mâu thuẫn.
TH2: [M]f[/M] chỉ nhận giá trị dương cũng mâu thuẫn 
Vậy [M]f[/M] chỉ nhận giá trị không dương .
Như thế [M]f(f(x))=0[/M] với mọi [M]x[/M]
suy ra [M]f(0)=f(f(f(x)))=0[/M]
Ta có [M]f(x+y) \le yf(x)[/M]
Cho [M]y=-x[/M] suy ra [M]-xf(x) \ge 0[/M] nên [M]f(x)[/M] không âm với [M]x[/M] không âm 
Suy ra [M]f(x)=0[/M] với [M]x[/M] không âm.

C2 Bài 3: $f(x+y) \leq yf(x)+f(f(x)). $ (*)
+ Cho $y=0 $, có $f(x)\leq f(f(x)) \; $ (1).
+ Cho $y=f(x)-x $, có $f(f(x)) \leq f(x)[f(x)-x]+f(f(x)) $, hay là $0 \leq f(x)[f(x)-x] \; $ (2).
a) Ta sẽ chứng minh $f(x)\leq 0 $ với mọi $x $. Thật vậy, nếu có $x $ sao cho $f(x)>0 $ thì
$f(f(y))=f(f(y)-x+x) \leq [f(y)-x]f(x)+f(f(x)) = [f(y-x+x)-x]f(x)+f(f(x)) \leq [(y-x)f(x)+f(f(x))-x]f(x)+f(f(x)). $
Ta có $f(f(y))\leq y.f^2(x)+h(x) $ với mọi $y $ (3) ($h(x) $ là hàm xác định từ biểu thức trên).
Chú ý là: từ (1) và (2), ta thấy $0 \leq f(f(x))[f(f(x))-f(x)] $ và $f(x)\leq f(f(x)) $, nên nếu $f(f(x))<0 $ thì phải có $f(f(x))=f(x) $.
Như vậy, từ (3) suy ra: khi $y $ tới âm vô cùng thì $f(y)=f(f(y)) $ cũng tới âm vô cùng.
Thay $y=-x $ vào (*), có $f(0)\leq -xf(x)+f(f(x))=f(x)(-x+1) $ (khi $x $ đủ nhỏ thì $f(x)=f(f(x)) $). Cho $x $ tới âm vô cùng thì vế phải cũng tới âm vô cùng, trong khi vế trái cố định. Mâu thuẫn.
b) Ta chứng minh rằng tồn tại N để với mọi $x<N $ thì $f(x)=0. $
Phản chứng, nếu không tồn tại N thì từ ý a), ta suy ra tồn tại dãy $\{x_n\} $ dần tới âm vô cùng sao cho $f(x_n)<0 $. Như vậy, từ (2) ta suy ra $f(x)-x\leq 0 $.
Lại thay $y=-x $ vào (*), có $f(0) \leq -xf(x)+f(f(x)) \leq -xf(x) $. Chọn dãy $\{x_n\} $ tiến tới âm vô cùng thay vào, ta có vế trái cố định, vế phải ra âm vô cùng. Mâu thuẫn.
c) Xét $x<N $, ta có $0 \geq f(0)=f(f(x)) \geq f(x) =0 $. Vậy $f(0)=0 $.
d) Xét $x<0 $, có $0=f(0)=f(-x+x) \leq -xf(x)+f(f(x))\leq -xf(x) \leq 0 $. Dấu bằng xảy ra khi $f(x)=0 $.
KL: $f(x)=0 $ với mọi $x\leq 0 $.  

Bài 2:
Ta gọi một đường thằng $d $ đi qua điểm $A $ và một điểm $B $ là tốt với $A $ nếu quay $d $ quanh $A $ một góc $\alpha $ nhỏ (chiều quay kim đồng hồ) thì hai nửa mặt phẳng chứa số điểm chênh lệch nhau không quá $1 $.

Nhận xét 1: Mỗi điểm $A $ thì có ít nhất $1 $ đường thẳng tốt đi qua. 

Bây giờ ta sẽ chứng minh là với cách chọn điểm $A $ bất kì và đường thẳng $ l $ là đường thẳng tốt qua $A $ 

Nhận xét 2: Giả sử tại bước thứ $k $, ta quay tại điểm $A $ và đường thẳng $l $ là đường thẳng tốt thì ta sẽ đến tới điểm $B $, đường thẳng $l $ sẽ trở thành đường thẳng $BA $ và $BA $ là tốt
Chứng minh:

Thật vậy ta giả sử $l $ chứa $2 $ điểm $A $ và $X $( song song với đường ngang). Số điểm nằm ở mặt phẳng phía trên là $a $, phía dưới là $b $. Ta có do $l $ tốt nên $|a+1-b|\le 1 $. Nếu $B $ ở mặt phẳng phía trên thì khi quay đường $BA $ một góc $\alpha $ nhỏ chia mặt phẳng thành hai phần có số điểm chênh nhau là $|a+1-b|\le 1 $. Nếu $B $ ở phía dưới thì cũng dễ thấy khi quay $BA $ quanh $B $ một góc $\alpha $ nhỏ đi thì số điểm chênh lệch ở hai nửa mặt phẳng là $|a+1-b|\le 1 $. Do đó $BA $ là đường thẳng tốt. Nhận xét được chứng minh.

Nhận xét 3: Mỗi điểm $X $ thuộc $S $ sẽ được ghé đến ít nhất một lần không phụ thuộc vào điểm đầu ( và do đó được ghé đến vô hạn lần).

Ta biết rằng quá trình quay đường thẳng $l $ theo chiều kim đồng hồ là một quá trình vô hạn lần và sẽ có lúc đường thẳng $l $ song song với đường thẳng tốt $d $ đi qua $X $ và điểm $M $. Nếu $l $ không trùng $d $, xét điểm cuối cùng mà $l $ đi qua trước khi song song với $X $ là $N,Y $. Dễ thấy là $M,X $ khác với $N,Y $Tuy nhiên dẽ thấy là sẽ có ít nhất một trong hai đường thẳng $NY $ và $MX $ không tốt. ( chỉ cần so sánh số điểm chênh lệch ở các nửa mặt phẳng là ok)

Vậy bài toán được chứng minh  

 

 



#17
toanhoc2017

toanhoc2017

    Thiếu úy

  • Banned
  • 628 Bài viết

Bài 1: Cho $a_0 < a_1 < a_2 \ldots$ là một dãy vô hạn số nguyên dương. Chứng minh rằng tồn tại duy nhất $n\geq 1$ sao cho

\[a_n < \frac{a_0+a_1+a_2+\cdots+a_n}{n} \leq a_{n+1}.\]

 

Bài 2: Cho $n \ge 2$ là một số nguyên. Xét một bàn cờ kích thước $n \times n$ chứa $n^2$ hình vuông đơn vị. Một bộ $n$ quân cờ được gọi là hòa bình nếu mỗi hàng và mỗi cột chứa đúng 1 quân. Tìm số nguyên dương $k$ lớn nhất, sao cho, với mỗi bộ $n$ quân cờ hòa bình, tồn tại một hình vuông $k \times k$ không chứa bất cứ quân cờ nào trong $k^2$ ô vuông của nó.

 

Bài 3: Tứ giác lồi $ABCD$ có $\angle ABC = \angle CDA = 90^{\circ}$. Điểm $H$ là chân đường vuông góc từ $A$ xuống $BD$. Các điểm $S$ và $T$ nằm trên $AB$ và $AD$, tương ứng, sao cho $H$ nằm ở miền trong $SCT$ và

 

$\angle CHS - \angle CSB = 90^{\circ}, \quad \angle THC - \angle DTC = 90^{\circ}.$ Chứng minh rằng $BD$ là tiếp tuyến đường tròn ngoại tiếp tam giác $TSH$ 

Harry Potter is offline   

 

 

Bài 1: cũng không phải quá đơn giản.
Nhận xét: nếu có N mà $\frac{x_0+x_1+..+x_N}{N}\leq x_{N} $ (1) thì mọi n>N-2 đều có $\frac{x_0+x_1+..+x_n}{n}\leq x_{n}<x_{n+1} $ (quy nạp đơn giản là có điều này)
Điều này gợi ý cho ta việc nếu chỉ số nhỏ nhất thỏa mãn (1) là k>1, thì chỉ số thỏa đề bài là k-1.
Tồn tại chỉ số k này không khó chỉ ra, chọn $n>x_0 $ thì hiển nhiên thỏa mãn (1).
Hơn nữa, hiển nhiên có k>1. Do vậy có đpcm.  

Bài 3:  
Ta sẽ dựng ra hai điểm S,T theo một cách "không liên quan".
1, Dựng S,T. Vẽ trung trực của BD cắt CH tại O, ta xét đường tròn tâm O, bán kính BO, nó cắt lại CH tại X trên tia CH. Qua X vẽ tiếp tuyến với đường tròn (O) cắt AB,AD tại S và T.
Như vậy ta thấy $\widehat{BCS}=\widehat{BXS}=\widehat{XDB} $, từ đó suy ra DX và CS cắt nhau tại K nằm trên (w) ngoại tiếp ABCD.
Tương tự BX và CT cắt nhau tại R nằm trên (w).
Chỉ dùng góc là có thể thấy KR và ST song song.
2. Ta sẽ chỉ ra rằng HS và HT lần lượt vuông góc BX,XD.
Để làm điều này, ta cần bổ đề sau:
Cho hình thang KRTS, KS cắt RT tại C. Vẽ CY vuông góc KR, X là giao của CY và ST. Khi đó, lấy M trên CX để $MT \perp XS $thì $MS \perp XR $.
Thật vậy: $RS^2-RM^2+XM^2-SX^2=RS^2-(KM^2+RY^2-YS^2)+XM^2-SX^2 $
Chú ý là đã có : $MT \perp XS $, nên $KM^2-KT^2+XT^2-XM^2=0 $
Thay vào, suy ra ta cần có $SR^2-KT^2=RY^2+SX^2-KY^2-XT^2 $ ( cái này không khó để chỉ ra trong hình thang KSTR có KS cắt RT tại C và X,Y là chân các đường cao từ C)
Như vậy, ta đã lấy được $M $ mà $MT,MS $ lần lượt vuông góc $XD, XB. $
Do vậy $MB^2-MD^2= (BS^2+XM^2-SX^2)-(DT^2+XM^2-XT^2)= (XC^2-CB^2)-(XC^2-CD^2)=CD^2-CB^2=AB^2-AD^2 $, vậy $AM $ vuống góc $BD $, nói cách khác $M $ là giao của $AH $ và $CH $, do đó $M $ trùng $H $.
3, Vậy có các điều kiện $HS,HT $ lần lượt vuông góc $XB,XD $.
Ta suy ra $\widehat{CHS}=90+\widehat{XSH}=90+\widehat{BXC}=90 +\widehat{BSC} $, dễ thấy cách xác định "S của đề bài là duy nhất" nên $S $ được dựng như trên. Tương tự với $T $.
Cuối cùng $\widehat{BMS}=90-\widehat{HBX}=90-\widehat{HBR}=90-\widehat{RCD}=\widehat{CTD}=\widehat{CTS} $
Như thế thì $BH $ tiếp xúc với $(HST) $, và ta có đpcm.

 

Ta sẽ chỉ ra rằng với $k^2<n<(k+1)^2+1 $ thì với bàn cờ ô vuông cạnh n, k là số lớn nhất thỏa mãn.
Thật vậy, ta chỉ cần xét $n=(k+1)^2 $. Lúc này, chia bàn cờ thành $(k+1)^2 $ hình vuông cạnh k+1. 
Xét bộ quân xe mà ở mỗi hình vuông có đúng 1 quân xe, hình vuông ở hàng thứ i, cột thứ j thì quân xe được đặt vào hàng thứ i, cột thứ j của hình vuôn đó.
Dễ thấy trong cấu hình này, ta không thể tìm ra hình vuông nào cạnh k+1, không chứa bất kỳ quân xe nào. Tương tự suy ra với các số n thuộc đoạn trên thì k là số lớn nhất thỏa mãn.
Ta chứng minh, với các n trên, thì luôn tìm được một hình vuông cạnh k thỏa mãn. Thật vậy: Đặt $n=k^2+r $.
Giả sử có cách xếp các quân xe mà không hình vuông cạnh k nào là trống. 
Xét hình vuông lớn cạnh $k^2 $ ở góc trên bên trái bàn cờ, nó được chia thành $k^2 $ hình vuông cạnh k. Nếu mỗi hình vuông này đều không trống thì suy ra trong hình vuông cạnh $k^2 $ này phải có ít nhất $k^2 $ quân cờ. Và hơn nữa, trong hình vuông cạnh $k^2 $ này cũng chỉ có tối đa $k^2 $ quân cờ ( do điều kiện mỗi hàng một quân)
Xét r cột cuối cùng bên phải, và r hàng cuối cùng bên trái, ta còn đúng r quân cờ để bỏ vào. Mà để mỗi hàng có đúng 1 quân, mỗi cột đúng 1 quân thì bắt buộc phải có r quân trong hình vuông cạnh r ở góc dưới bên phải của bảng.
Như vậy xét hình chữ nhật $k^2 $ nhân r ở góc trên bên phải của bảng thì nó không chứa bất kỳ quân cờ nào. Nếu $r>k $, ta có ngay một hình vuôn cạnh k trống.
Nếu $r<k $, xét hình chữ nhật k nhân $k^2 $ ở góc trên bên phải của bảng. Theo giả sử phản chứng thì hình chữ nhật này có ít nhất k quân cờ. Mặt khác, nó lại chỉ gồm k-r cột có thể chứa quân cờ, tức là nó có tối đa k-r quân cờ. Hai điều này mâu thuẫn với nhau, từ đó ta suy ra đpcm.

 

Dưới đây là một lời giải khác cho bài hình:
Giả sử trung trực $SH$ cắt $AH$ ở $I$, cắt $AB$ kéo dài ở $X$; trung trực $TH$ cắt $AH$ ở $J$, cắt $AD$ kéo dài ở $Y$. 

Gọi $M,N,P$ lần lượt là trung điểm $HS,HT,HC$ và $O$ là trung điểm của $AC$, cũng là tâm của $(ABCD)$. Ta có $PO\parallel AH,AH\bot BD\Rightarrow PO\bot BD$ nên $PB=PD$.

Đặt $\angle CSX=\alpha ,\angle CTY=\beta $ thì theo giả thiết, ta có $\angle CHS=90{}^\circ +\alpha ,\angle CHT=90{}^\circ +\beta $. 

Mặt khác, gọi ${X}'$ là tâm của $(HSC)$ thì $\angle C{X}'S=360{}^\circ -2(90{}^\circ +\alpha )=180{}^\circ -2\alpha $ nên ta được $\angle {X}'SC=\angle {X}'CS=\alpha =\angle XSC$, suy ra ${X}'\in SB$ hay $X\equiv {X}'$. Từ đó, ta có $X,Y$ chính là tâm của các đường tròn ngoại tiếp tam giác $CHS,CHT$. 

Theo định lý Menelaus thì \[\frac{IA}{IH}\cdot \frac{MH}{MS}\cdot \frac{XS}{XA}=1\Rightarrow \frac{IA}{IH}=\frac{XA}{XS}.\] Tương tự, ta cũng có $\dfrac{JA}{JH}=\dfrac{YA}{YT}$. Do đó, để $(HST)$ tiếp xúc với $BD$ thì tâm của nó phải nằm trên $AH$ hay nói cách khác, $I\equiv J$ và ta cần có $\dfrac{XA}{XS}=\dfrac{YA}{YT}\Leftrightarrow \dfrac{XA}{YA}=\dfrac{XS}{YT}$.

Chú ý rằng các tứ giác $CPBX,CPDY$ nội tiếp. Theo định lý sin thì 
$$\dfrac{XA}{YA}=\dfrac{XC}{YC}\Leftrightarrow \dfrac{\sin AYX}{\sin AXY}=\dfrac{\sin CYX}{\sin CXY}\Leftrightarrow \dfrac{\sin PCB}{\sin PCD}=\dfrac{\sin PBC}{\sin PDC}\Leftrightarrow \dfrac{\sin PCB}{\sin PBC}=\dfrac{\sin PCD}{\sin PDC}.$$ Ta cũng có $$\frac{\sin PCB}{\sin PBC}=\frac{PB}{PC}=\frac{PD}{PC}=\frac{\sin PCD}{\sin PDC}.$$ Từ đó, ta có đpcm

 

 

.  Chi tiết bài 1 xem thế nào:

+) Giả sử không tồn tại $n $ thỏa $a_n < \frac{a_0+a_1+a_2+\cdots+a_n}{n} \leq a_{n+1} $
Hay với mọi $n $ thì $\left[ \begin{array}{ll} a_0+a_1+a_2+\cdots+a_n > na_{n+1} \\ a_0+a_1+a_2+\cdots+a_n \le na_n \end{array} \right $
Giả sử $n_0 $ là nhỏ nhất thỏa $a_0+a_1+a_2+\cdots+a_{n_0} > n_0a_{n_0+1} $.
$\to $ $a_0+a_1+a_2+\cdots+a_{n_0+1} > (n_0+1) a_{n_0+1} $ 
$\to $ $a_0+a_1+a_2+\cdots+a_{n_0+1} > (n_0+1) a_{n_0+2} $
Vậy $a_0+a_1+a_2+\cdots+a_n \le na_n \forall n \le n_0 $ Sai ngay với $n=1 $ $\to n_0=1 $ 
Vậy:
$a_0+a_1>a_2 $
$a_0+a_1+a_2>2a_3 $
......
$a_0+a_1+a_2+a_3+...+a_n>na_{n+1} $ ($n $ tiến đến vô cùng)
Cộng vế với vế suy ra $na_0+na_1>na_{n+1} $
Vô lý do $n $ tiến đến vô cùng 
Vậy tồn tại $n $ thỏa $a_n < \frac{a_0+a_1+a_2+\cdots+a_n}{n} \leq a_{n+1} $


+) Giờ ta chứng minh tồn tại suy nhất:
Giả sử tồn tại $n < m $ đồng thời thỏa mãn :
$a_n < \frac{a_0+a_1+a_2+\cdots+a_n}{n} \leq a_{n+1} $ (*)
$a_m < \frac{a_0+a_1+a_2+\cdots+a_m}{m} \leq a_{m+1} $ (**)

Từ (*): $ a_0+a_1+a_2+\cdots+a_n \le na_{n+1} $
$\to a_0+a_1+a_2+\cdots+a_{n+1} \le (n+1)a_{n+1} $
$\to a_0+a_1+a_2+\cdots+a_{n+2} \le (n+1)a_{n+1} + a_{n+2}< (n+2)a_{n+2} $
Lặp lại chu trình trên đến khi $n=m $ ta sẽ được;
$a_0+a_1+a_2+\cdots+a_{m} < ma_{m} $ Vô lý với điều giả sử (**)



#18
toanhoc2017

toanhoc2017

    Thiếu úy

  • Banned
  • 628 Bài viết

Trò chơi đoán kẻ nói dối là một trò chơi giữa hai người chơi $A$ và $B$. Quy tắc của trò chơi phụ thuộc vào hai số nguyên dương $k$ và $n$ mà cả hai người chơi đều đã biết trước.

Bắt đầu trò chơi, $A$ sẽ chọn các số nguyên $x$ và $N$ với $1 \le x \le N$. $A$ giữ bí mật số $x$ và nói số $N$ cho $B$. $B$ sẽ cố thu nhận thông tin về số $x$ bằng cách hỏi $A$ các câu hỏi như sau : mỗi câu hỏi bao gồm việc $B$ xác định một tập $S$ tùy ý các số nguyên dương (có thể là một tập đã được nhắc đến trong câu hỏi trước đó) và hỏi $A$ xem $x$ có thuộc $S$ hay không. Sau mỗi câu hỏi, $A$ phải trả lời hoặc không, nhưng có thể nói dối bao nhiêu lần tùy thích, chỉ có điều là phải trả lời đúng ít nhất một trong số $k+1$ câu hỏi liên tiếp.

Sau khi $B$ đã hỏi xong, $B$ phải chỉ ra một tập $X$ có tối đa $n$ số nguyên dương. Nếu $x \in X$, $B$ thắng; nếu ngược lại, $B$ thua. Chứng minh rằng :

  1. Nếu $n \ge 2^k$, $B$ có thể đảm bảo một chiến thắng.
  2. Với mọi $k$ đủ lớn, tồn tại một số nguyên $n \ge 1.99^k$ sao cho $B$ không thể đảm bảo có một chiến thắng.

Giả sử đã có $x \in S = \{1, ..., n\}$ với $n > 2^k$, ta chứng minh rằng sau một số câu hỏi, có thể tìm được tập con $S' \subset S$ sao cho $|S'| < |S|$ và chắc chắn $x \in S'$. Quá trình này sẽ dừng khi $|S| \leq 2^k$, và do đó suy ra được điều cần chứng minh.

 

Thật vậy, vì $|S| > 2^k$ nên tồn tại song ánh $f$ giữa một tập con thật sự $T$ của $S$ và tập hợp các chuỗi nhị phân độ dài $k$. Gọi $m$ là một phần tử nằm trong $S$ nhưng ko nằm trong $T$. Đồng thời ký hiệu $T_i$ là nghịch ảnh của tập hợp các chuỗi nhị phân có bit ở vị trí $i$-th là $0$, đối với $f$. Ta thực hiện các câu hỏi sau:

 

1. Hỏi liên tục các câu với tập hợp $\{m\}$. Nếu sau $k + 1$ câu hỏi như vậy mà ta đều nhận được đáp án "có" thì vì trong các câu này phải có ít nhất 1 câu $A$ nói thật nên có thể kết luận $X = \{m\}$, hết chuyện.

 

2. Ngược lại, giả sử tại câu hỏi thứ $i$-th ta có câu trả lời "không". Từ câu $i + 1$ đến câu $i + k$, ta lần lượt hỏi với các tập hợp $T_i$ như đã mô tả ở trên. Vì $i...i + k$ là $k + 1$ câu hỏi liên tiếp nên trong số chúng, có ít nhất 1 câu đáng tin. Thế tức là nếu ta kết hợp tất cả các điều kiện ngược với các câu trả lời thì dựa vào đó, tập hợp thu được sẽ vẫn chứa $x$. Ký hiệu $res(T_i) = T_i$ nếu câu trả lời ứng với câu hỏi $T_i$ là "có" và $res(T_i) = \overline{T_i}$ nếu câu trả lời này là "không" (ở đây $\overline{T_i}$ là tập hợp các chuỗi nhị phân độ dài $k$ và có bit ở vị trí $i$-th là $1$, hợp với các phần tử còn lại trong $S$, ngoài $m$ và các phần tử trong các $T_j$). Ta tìm giao của các tập hợp $\overline{res(T_i)} \cap (\bigcup T_i)$. Rõ ràng mỗi tập hợp kiểu này cho ta một mô tả về phần tử $x$ nằm trong giao của chúng (i.e. $x$ có bit thứ $i$-th là $0$ hay $1$). Vậy nên giao này khác rỗng (và tất nhiên là không phủ hết $(\bigcup T_i)$, vì bản thân các $\overline{res(T_i)}$ cũng không phủ được!). Do vậy, phần tử $x$ cần tìm mất đi ít nhất $1$ khả năng, và đó là điều ta đang muốn  



#19
toanhoc2017

toanhoc2017

    Thiếu úy

  • Banned
  • 628 Bài viết

Bài 4.

Tìm tất cả các hàm số $f : \mathbb{Z} \to \mathbb{Z}$ sao cho với mọi $a+b+c=0$ thì
$$ f(a)^2+f(b)^2+f(c)^2 = 2f(a)f(b) + 2f(b)f(c) + 2f(c)f(a). $$


 



#20
toanhoc2017

toanhoc2017

    Thiếu úy

  • Banned
  • 628 Bài viết

Cho tam giác $ABC$ có $\widehat{ACB} = 90^\circ$ và $D$ là chân đường cao tương ứng với đỉnh $C$. Gọi $X$ là một điểm trong của đoạn thẳng $CD$. Gọi $K$ là điểm trên đoạn thẳng $AX$ sao cho $BK=BC$. Tương tự, gọi $L$ là điểm trên đoạn thẳng $BX$ sao cho $AL=AC$. Gọi $M$ là giao điểm của $AL$ và $BK$. Chứng minh rằng $MK=ML$.
 






1 người đang xem chủ đề

0 thành viên, 1 khách, 0 thành viên ẩn danh